This site is 100% ad supported. Please add an exception to adblock for this site.

Surgery Questions USMLE 2

Terms

undefined, object
copy deck
A 14-year-old boy is hit over the right side of the head with a baseball bat. He loses consciousness for a few minutes, but recovers promptly and continues to play. One hour later he is found unconscious in the locker room. His right pupil is fixed and d
Dx: Acute epidural hematoma (probably right side)

Diagnostic Test: CT scan

Treatment: Emergency surgical decompression (craniotomy)

Good prognosis if treated, fatal within hours if it is not.
A 32-year-old male is involved in a head-on, high-speed automobile collision. He is unconscious at the site, regains consciousness briefly during the ambulance ride and arrives at the E.R. in deep coma, with a fixed, dilated right pupil.
Dx?
Di
Dx: Acute Subdural hematoma

Diagnostic Test: CT scan
(Also need to check cervical spine!)

Treatment: Emergency craniotomy

poor prognosis because of brain injury
A 77-year-old man becomes “senile” over a period of three or four weeks. He used to be active and managed all of his financial affairs. Now he stares at the wall, barely talks and sleeps most of the day. His daughter recalls that he fell from a horse
Dx: Chronic subdural hematoma.

Diagnostic Test: CT scan

Treatment: Surgical decompression (craniotomy)

Spectacular improvement expected
A car hits a pedestrian. He arrives in the ER in coma. He has⬦(raccoon eyes⬦ or clear fluid dripping from the nose⬦or clear fluid dripping from the ear⬦or ecchymosis behind the ear)⬦
Dx?
Diagnostic Test?
Tx?
Dx: Base of the skull fracture.

Diagnostic Test: CT scan and cervical spine X-Rays.

Tx: needs neurosurgical consult and antibiotics
A 45-year-old man is involved in a high-speed automobile collision. He arrives at the ER in coma, with fixed dilated pupils. He has multiple other injuries (extremities, etc). His blood pressure is 70 over 50, with a feeble pulse at a rate of 130. What i
significant blood loss to the outside (could be scalp laceration), or inside (abdomen, pelvic fractures)⬦not from neurological injury
A 22-year-old gang member arrives in the E.R. with multiple guns shot wounds to the abdomen. He is diaphoretic, pale, cold, shivering, anxious, asking for a blanket and a drink of water. His blood pressure is 60 over 40. His pulse rate is 150, barely per
Dx: Hypovolemic shock

Management: Big bore IV lines, Foley catheter and I.V. antibiotics.

Tx: Ideally Exploratory Lap immediately for control of bleeding, and then fluid and blood administration.
A 22-year-old gang member arrives in the E.R. with multiple guns shot wounds to the chest and abdomen. He is diaphoretic, cold, shivering, anxious, asking for a blanket and a drink of water. His blood pressure is 60 over 40. His pule rate is 150, barely
Dx: Pericardial tamponade

Diagnostic test: No X-Rays needed, this is a clinical diagnosis!
Do Pericardial window.

Tx: If positive, follow with Thoracotomy, and then Exploratory Lap.
A 22-year-old gang member arrives in the E.R. with a single gunshot wound to the precordial area. He is diaphoretic, cold, shivering, anxious, asking for a blanket and a drink of water. His blood pressure is 60 over 40. His pule rate is 150, barely perce
Dx: Pericardial Tamponade

Management: Exploratory Lap

(when the location of the wound strongly suggests pericardial tamponade, emergency thoracotomy might be done right away without prior pericardial window)
A 22-year-old gang member arrives in the E.R. with multiple gun shot wounds to the chest and abdomen. He has labored breathing is cyanotic, diaphoretic, cold and shivering. His blood pressure is 60 over 40. His pulse rate is 150, barely perceptible. He i
Dx: Tension pneumothorax

Management:
1. Immediate big bore IV catheter placed into the right pleural space (2nd intercostal midclavicular)
2. followed by Chest Tube to the right side, Immediately!

(Watch out for trap that offers chest X-Ray as an option. This is a clinical diagnosis, and patient needs that chest tube now. He will die if sent to X-Ray.)

Tx: Exploratory lap will follow
A 72 year old man who lives alone calls 911 saying that he has severe chest pain. He cannot give a coherent history when picked up by the EMT, and on arrival at the ER he is cold and diaphoretic and his blood pressure is 80 over 65. He has an irregular,
Dx: Cardiogenic shock, from massive MI

Management: verify high CVP. EKG, enzymes, coronary care unit etc. Do not drown him with enthusiastic fluid “resuscitation”, but use thrombolytic therapy if offered
A 17 year old girl is stung by a swarm of bees⬦or a man of whatever age breaks out with hives after a penicillin infection⬦or a patient undergoing surgery under spinal anesthetic⬦eventually develop BP of 75 over 25, pulse rate of 150, but they look
Dx: Vasomotor shock
(massive vasodilation, loss of vascular tone)

Management: Vasoconstrictors and Volume replacement as needed
A 25-year-old man is stabbed in the right chest. He is moderately short of breath, has stable vital signs. No breath sounds on the right. Resonant to percussion.
Dx?
Diagnostic Test?
Tx (specific)?
Dx: Plain pneumothorax

Diagnostic Test: There is time to get a chest X-Ray if the option if offered

Treatment: Chest tube to underwater seal and suction, high in the pleural cavity
A 25-year-old man is stabbed in the right chest. He is moderately short of breath, has stale vital signs. No breath sounds on at the base on the right chest, faint distant breath sounds at the apex. Dull to percussion.
Dx?
Diagnostic Test?
Dx: Hemothorax

Diagnostic Test: Chest X-Ray

Treatment: Chest tube on the right, at the base of the pleural cavity
A 25-year-old man is stabbed in the right chest. He is moderately short of breath, has stable vital signs. No breath sounds on at the base on the right chest, faint distant breath sounds at the apex. Dull to percussion. A chest tube placed at the right p
Dx: Hemothorax

Further treatment: The point of this one is that most hemothoraxes do not need exploratory surgery. Bleeding is from lung parenchyma (low pressure), stops by itself. Chest tube is all that is needed.

Key clue: little blood retrieved, even less afterwards
A 25-year-old man is stabbed in the right chest. He is moderately short of breath, has blood pressure is 95 over 70, pulse rate of 100. No breath sounds on at the base on the right chest, faint distant breath sounds at the apex. Dull to percussion. A che
Dx: Hemothorax

Further treatment: The rare exception who is bleeding from a systemic vessel (almost invariably intercostal) will need Thoracotomy to ligate the vessel
A 25-year-old man is stabbed in the right chest. He is moderately short of breath, has stable vital signs. No breath sounds on the right. Resonant to percussion at the apex of the right chest, dull at the base. Chest X-Ray shows one single, large air-flu
Dx: Hemo-pneumothorax

Tx: Chest tube, surgery only if bleeding a lot
A 33-year-old lady is involved in a high-speed automobile collision. She arrives at the E.R. gasping for breath, cyanotic at the lips, with flaring nostrils. There are bruises over both sides of the chest, and tenderness suggestive of multiple fractured
Dx: Tension Pneumothorax

Where is the penetrating trauma? The fractured ribs can act as a penetrating weapon.

Management: Chest Tube to the left immediately!
A 54-year-old lady crashes her car against a telephone pole at high speed. On arrival at the E.R. she is in moderate respiratory distress. She has multiple bruises over the chest, and multiple site of point tenderness over the ribs. X-Rays show multiple
Dx: Flail Chest
(paradoxical breathing)

to OR:
prophylactic Bilateral Chest Tubes
(because she is at high risk to develop tension pneumothorax when under the positive pressure breathing of the anesthetic)

not well:
Intubate and give Positive Pressure ventilation
(Flail chest is usually assoc w/ pulmonary contusion, leading to inadequate respiration from pain)
A 54-year-old lady crashes her car against a telephone pole at high speed. On arrival at the E.R. she is breathing well. She has multiple bruises over the chest and multiple sites of point tenderness over the ribs. X-Rays show multiple rib fractures on b
Dx: Pulmonary contusion.

It does not always show up right away, may become evident one or two days after the trauma.

Management:
1. Fluid restriction (using colloids) and diuretics,
2. Respiratory support:
(intubation, mechanical ventilation and PEEP if needed)
A 54-year-old lady crashes her car against a telephone pole at high speed. On arrival at the E.R. she is breathing well. She has multiple bruises over the chest, and is exquisitely tender over the sternum at a point where there is a crunching feeling of
Dx: Sternal fracture
(but the point is that she is at high risk for myocardial contusion and for traumatic rupture of the aorta)

Further tests:
Most important:
1. CT scan
2. Transesophageal echo
(or arteriogram looking for aortic rupture)

Also work-up for MI:
1. EKG
2. Cardiac enzymes
A 53-year-old man is involved in a high-speed automobile collision. He has moderate respiratory distress. Physical exam shows no breath sounds over the entire left chest. Percussion is unremarkable. Chest X-Ray shows air fluid levels in the left chest.
Dx: Diaphragmatic rupture
(It is always on the left)

Management: Surgical repair
A motorcycle daredevil attempts to jump over the 12 fountains in front of Caesar’s Palace Hotel in Las Vegas. As he leaves the ramp at very high speed his motorcycle turns sideways and he hits the retaining wall at the other end, literally like a rag d
Dx: traumatic rupture of the aorta

(King size trauma, fracture of a hard-to-break bone...it could be first rib, scapula or sternum...and the tell-tale hint of widened mediastinum)

Diagnostic Test: Arteriogram (aortogram)

Treatment: Emergency surgical repair
A 34-year-old lady suffers severe blunt trauma in a car accident. She has multiple injuries to her extremities, has head trauma and has a pneumothorax on the left. Shortly after initial examination it is noted that she is developing progressive subcutane
Dx: Traumatic rupture of the trachea or major bronchus

Additional findings: Chest X-Ray would confirm the presence of air in the tissues

Diagnostic test: Fiberoptic bronchoscopy
(to confirm diagnosis and level of injury and to secure an airway)

Tx: Surgical repair
A 26-year-old lady has been involved in a car wreck. She has fractures in upper extremities, facial lacerations and no other obvious injuries. Chest X-Ray is normal. Shortly thereafter she develops hypotension, tachycardia and dropping hematocrit. Her CV
Dx: Abdominal bleed

Diagnostic test:
Patient is stable: CT scan
Unstable:
1. Diagnostic Peritoneal Lavage
or
2. Ultrasound in ER

Tx: Exploratory Lap
A 19 year old gang member is shot in the abdomen with a 38 caliber revolver. The entry wound is in the epigastrium, to the left of the midline. The bullet is lodged in the psoas muscle on the right. He is hemodynamically stable, the abdomen is moderately
Management:
A penetrating wound of the abdomen gets exploratory laparotomy every time.

preparations prior to surgery:
an indwelling bladder catheter, a big bore venous line for fluid administration and a dose of broad spectrum antibiotics.
A 19 year old gang member is shot once with a 38 caliber revolver. The entry wound is in the left mid-clavicular line, two inches below the nipple. The bullet is lodged in the left paraspinal muscles. He is hemodynamically stable, but he is drunk and com
Management:
The point here is to remind you of the boundaries of the abdomen. Although this sounds like a chest wound, it is also abdominal. The belly begins at the nipple line. The chest does not end at the nipple line, though. Belly and chest are not stacked up like pancakes, they are separated by a dome. This fellow needs all the stuff for a penetrating chest wound (chest X-Ray, chest tube if needed), plus the exploratory lap
A 27 year old intoxicated man smashes his car against a tree. He is tender over the left lower chest wall. Chest X-Ray shows fractures of the 8th, 9th and 10th ribs on the left. He has a blood pressure of 85 over 68 and a pulse rate of 128.
Dx?
Dx: Ruptured spleen

Management if Stable: CT Scan
(if he responds promptly to fluid administration, and does not require blood; further management in that case may well be continued observation with serial CT scans)

Management if “crashing”: Peritoneal Lavage or Sonogram followed by (Tx)Exploratory Laparotomy
A 27 year old intoxicated man smashes his car against a tree. He is tender over the left lower chest wall. Chest X-Ray shows fractures of the 8th, 9th and 10th ribs on the left. He has a blood pressure of 85 over 68 and a pulse rate of 128, which do not
Further Management:

administration of Pneumovax and some would also Immunize for Hemophilus Influenza B and Meningococcus
A 31 year old lady smashes her car against a wall. She has multiple injuries including upper and lower extremity fractures. Her blood pressure is 75 over 55, with a pulse rate of 110. On physical exam she has a tender abdomen, with guarding and rebound o
Dx: Blood (and possible feces) in the belly

Management: Exploratory lap
A 31 year old lady smashes her car against a wall. Her abdomen is tender with guarding and rebound tenderness present in all quadrants
Dx?
Management?
Dx: Ruptured bowel

Management: Exploratory lap, and repair of the injuries
A patient involved in a high speed automobile collision has multiple injuries, including a pelvic fracture. On physical exam there is blood in the meatus.
Dx? (2 possible)
Diagnostic test?
Dx: Bladder or Urethral injury
(pelvic fracture plus blood in the meatus)

Diagnostic test: Retrograde Urethrogram
(because urethral injury would be compounded by insertion of a Foley catheter)
A 19 year old male is involved in a severe automobile accident. Among many other injuries he has a pelvic fracture. He has blood in the meatus, scrotal hematoma and the sensation that he wants to urinate but can not do it. Rectal exam shows a “high rid
Dx: Posterior Urethral injury.

Diagnostic test: Retrograde Urethrogram

Management:
Suprapubic catheter
(and the repair is delayed 6 months)
A 19 year old male is involved in a motorcycle accident. Among many other injuries he has a pelvic fracture. He has blood in the meatus and scrotal hematoma. Retrograde urethrogram shows an anterior urethral injury.
Management?
Management: Anterior urethral injuries are repaired right away
A patient involved in a high speed automobile collision has multiple injuries, including a pelvic fracture. Insertion of a Foley catheter shows that there is gross hematuria.
Dx?
Diagnostic test?
Dx: Bladder injury
(Presumably there was no blood in the meatus to warn against the insertion of an indwelling catheter, and since the latter was accomplished without problem, the urethra must be intact)

Diagnostic test: Retrograde Cystogram
A patient involved in a high speed automobile collision has multiple injuries, including rib fractures and abdominal contusions. Insertion of a Foley catheter shows that there is gross hematuria, and retrograde cystogram is normal.
Dx?
Diagnost
Dx: Kidney injury
(Lower injuries have been ruled out)

Diagnostic test: CT scan

(They will not ask you for fine-judgment surgical decisions, but the rule is that traumatic hematuria does not need surgery even if the kidney is smashed.
They operate only if the renal pedicle is avulsed or the patient is exsanguinating)
A 35 year old male is about to be discharged from the hospital where he was under observation for multiple blunt trauma sustained in a car wreck. It is then discovered that he has microscopic hematuria.
Management?
Management: Gross traumatic hematuria in the adult always has to be investigated
A 4 year old falls from his tricycle. In the ensuing evaluation he is found to have microscopic hematuria.
Management?
Management: Microhematuria in kids needs to be investigated, as it often signifies congenital anomalies⬦particularly if the magnitude of the trauma does not justify the bleeding.
Start with a Sonogram
A 14 year old boy slides down a banister, not realizing that there is a big knob at the end of it. He smashes the scrotum and comes in to the E.R. with a scrotal hematoma the size of a grapefruit.
What should be the physician's concern?
Diagnos
Concern: The issue in scrotal hematomas is whether the testicle is ruptured or not.

Diagnostic test: Sonogram

Management: If ruptured, surgery will be needed. If intact, only symptomatic treatment
A 41 year old male presents to the E.R. reporting that he slipped in the shower and injured his penis. Exam reveals a large penile shaft hematoma with normal appearing glans.
Dx?
Tx?
Dx: Fracture of the tunica albuginea
(including the usual cover story given by the patient. These always happen during sexual intercourse with woman on top)

Tx: this is one of the few urological emergencies.
Surgical repair is needed
You get a phone call from a frantic mother. Her 7 year old girl spilled Drano all over her arms and legs. You can hear the girl screaming in pain in the background.
Management?
Management:
The point of this question is that chemical injuries – particularly alkalis-need copious, immediate, profuse irrigation.

Instruct the mother to do so right at home with tap water, for at least 30 minutes before rushing the girl to the E.R
While trying to hook up illegally to cable TV, an unfortunate man comes in contact with a high tension electrical power line. He has an entrance burn wound in the upper outer thigh and an exit burn lower down on the same side.
Tx?
What can occu
Management: Extensive surgical Debridement
(there is deep tissue destruction)

What can occur? Myoglobinemia
(leading to myoglobinuria and to Renal Failure)

Management:
1. lots of IV fluids,
2. Osmotic Diuretics (Mannitol),
3. Alkalinization of the urine
A man is rescued by firemen from a burning building. On admission it is noted that he has burns around the mouth and nose, and the inside of his mouth and throat look like the inside of a chimney.
Dx?
Diagnostic Test?
Management?
Dx: Inhalation burns

Diagnostic test: Bronchoscopy

Management: Respiratory support
A patient has suffered third degree burns to both of his arms when his shirt caught on fire while lighting the back yard barbecue. The burned areas are dry, white, leatherly anesthetic, and circumferential all around arms and forearms.
What is main
Problem: Circumferential burns
(The leatherly eschar will not expand, while the are under the burn will develop massive edema, thus circulation will be cut off or in the case of circumferential burns of the chest, breathing will be compromised)

Management: Compulsive monitoring of peripheral pulses and capillary filling.
Escharotomies at the bedside at the first sign of compromised circulation
A toddler is brought to the E.R. with burns on both of his buttocks. The areas are moist, have blisters and are exquisitely painful to touch. The story is that the kid accidentally pulled a pot of boiling water over himself.
what type of burn?
Dx: Second degree burn
(Note that in kids third degree is deep bright red, rather than white leatherly as in the adult)

Question: How did it really happen? Burns in kids always bring up the possibility of child abuse, particularly if they have the distribution that you would expect if you grabbed the kid by arms and legs and dunked him in a pot of boiling water.

Management: Silvadene cream.
Possibly reporting to authorities for child abuse
An adult male who weight “X” Kgs. Sustains second and third degree burns over ---whatever--- The burns will be depicted in a drawing, indicating what is second degree (moist, blisters, painful) and what is third degree (white, leatherly, anesthetic).
Management:
4cc per Kg. of body weight per percentage of burned area
(up to 50%)

(if pt is 70kg and 18% burned, then 70x4x18)

Fluid: Ringers Lactate

(half of the calculated dose goes in during first 8 hours)
A 42 year old lady drops her hot iron on her lap while doing the laundry. She comes in with the shape of the iron clearly delineated on her upper thigh. The area is white, dry, leatherly, anesthetic.
Tx?
Tx: Early excision and skin grafting
(in very small third degree burns)
A 22 year old gang leader comes to the E.R. with a small, 1 cm. deep sharp cut over the knuckle of the right middle finger. He says he cut himself with a screwdriver while fixing his car.
Dx?
Management?
Dx: The description is classical for a human bite. No, nobody actually bit him, he did it by punching someone in the mouth...and getting cut with the teeth that were smashed by his fist. The imaginative cover story usually comes with this kind of lesion.

Management: human bites are bacteriological the dirtiest that one can get. Rabies shots will not be needed, but surgical exploration by an orthopedic surgeon will be required.
A 65 year old West Texas farmer of Swedish ancestry has an indolent, raised, waxy, 1.2 cm skin mass over the bridge of the nose that has been slowly growing over the past three years. There are no enlarged lymph nodes in the head and neck.
Dx?
Dx: Basal cell carcinoma

Diagnostic test: Full thickness biopsy at the edge of the lesion (punch or knife)

Treatment: Surgical excision with clear margins, but conservative width
A 71 year old West Texas farmer of Irish ancestry has a non-healing, indolent, punched out, clean looking 2 cm ulcer over the left temple, that has been slowly becoming larger over the past three years. There are no enlarged lymph nodes in the head and n
Dx: Basal cell carcinoma

Diagnostic Test: Full thickness biopsy at the edge of the lesion (punch or knife)

Tx: Surgical excision with clear margins, but conservative width
A blond, blue eyed, 69 year old sailor has a non-healing, indolent 1.5 cm. ulcer on the lower lip, that has been present, and slowly enlarging for the past 8 months. He is a pipe smoker, and he has no other lesions or physical findings.
Dx?
Dia
Dx: Squamous cell carcinoma

Diagnostic test: Biopsy

Treatment: Surgical resection with wider (about 1 cm) clear margins.
Local radiation therapy is another option
A red headed 23 year old lady who worships the sun, and who happens to be full of freckles, consults you for a skin lesion on her shoulder that concerns her. She has a pigmented lesion that is asymmetrical, with irregular borders, of different colors wit
Dx: Melanoma or Dysplastic Nevus

Diagnostic test: full thickness biopsy at the edge of the lesion

- margin free local excision if superficial melanoma
(Clarks’ levels one or two, or under 0.75 mm)
- wide local excision with 2 or 3 cm margin if deep melanoma
A 35 year old blond, blue eyed man left his native Minnesota at age 18, and has been living the life as a crew member for a sailing yacht charter operation in the Caribbean. He has multiple nevi all over his body, but one of them has changed recently.
Dx: Melanoma

(Change in a pigmented lesion is the other tip off to melanoma...It may be growth, or bleeding, or ulceration, or change in color)

Management: Full-thickness biopsy at the edge of the lesion

- margin free local excision if superficial melanoma
(Clarks’ levels one or two, or under 0.75 mm)
- wide local excision with 2 or 3 cm margin if deep
A 44 year old man has unequivocal signs of multiple liver metastasis, but no primary tumor has been identified by multiple diagnostic studies of the abdomen and chest. The only abnormality in the physical exam is a missing toe, which he says was removed
Dx: Malignant Melanoma

(the alternate version has a glass eye, and history of enucleation for a tumor. No self-respecting malignant tumor would have this time interval, but melanoma will)

Diagnostic Test: full thickness biopsy at the edge of the lesion

- margin free local excision if superficial melanoma
(Clarks’ levels one or two, or under 0.75 mm)
- wide local excision with 2 or 3 cm. margin if deep melanoma
A 32 year old gentleman had a Clark’s level 5, 3.4 mm. Deep, melanoma removed from the middle of his back three years ago. He now has…(a tumor in a weird place, like his left ventricle, his duodenum, his ischiorectal area...anywhere!)
Dx?
Dx: Melanoma

(The point of this vignette is that invasive melanoma...it has to be deep...metastasizes to all the usual places [lymph nodes plus liver-lung-brain-bone] but it is also the all-time-champion in going to weird places where few other tumors dare to go)
An 18 year old lady has a firm, rubbery mass in the left breast that moves easily with palpation.
Dx?
Diagnostic Test?
Imaging technique for young patient?
Dx: Fibroadenoma

Diagnostic Test: Tissue diagnosis...(choices in order)
1. FNA; 2. Core Bx; 3. Excisional Bx

(The only safe answer, even if the presentation favors benign disease, is to get tissue diagnosis)

Sonogram is the only imaging technique suitable for the very young breast
A 27 year old immigrant from Mexico has a 12 x 10 x 7 cm. mass in her left breast. It has been present for seven years, and slowly growing to its present size. The mass is firm, rubbery, completely movable, is not attached to chest wall or to overlying s
Dx: Cystosarcoma Phyllodes

(basically same presentation as Fibroadenoma, but >25yo)

Diagnostic test: given the size best done with core or incisional biopsy

(no need for axillary node dissection w/ phyllodes...mets is rare)
A 35 year old lady has a ten year history of tenderness in both breasts, related to menstrual cycle, with multiple lumps on both breasts that seem to “come and go” at different times in the menstrual cycle. Now has a firm, round, 2 cm. mass that has
Dx: Fibrocystic disease

Diagnostic test: Aspiration of the Cyst

(tissue diagnosis [i.e: biopsy] becomes impractical when there are lumps every month)

If the mass goes away and the fluid aspirated is clear, that’s all.
If the fluid is bloody it goes to cytology.
If the mass does not go away, or recurs she needs biopsy.
(Answers that offer mammogram or sonogram in addition to the aspiration would be OK, but not as the only choice)
A 34 year old lady has been having bloody discharge from the right nipple, on and off for several months. There are no palpable masses.
Dx?
Diagnostic Test?
if test is inconclusive?
Dx: Intraductal papilloma

Diagnostic test: Mammogram
(the way to detect breast cancer that is not palpable)

(If negative, one may still wish to find an resect the intraductal papilloma to provide symptomatic relief. Resection can be guided by galactogram, or done as a retroareolar exploration)
A 26 year old lactating mother has cracks in the nipple and develops a fluctuating, red, hot, tender mass in the breast, along with fever and leukocytosis.
Dx?
Management?
Dx: Abscess

(However, only lactating breasts are “entitled” to develop abscesses. On anybody else, a breast abscess is a cancer until proven otherwise.)

Management: Incision and Drainage

(if an option includes drainage with biopsy of the abscess wall, go for that one)
A 49 year old has a firm 2cm mass in the right breast that has been present for 3 months.
Dx?
Management?
Dx: This could be anything.

(Age is the best determinant for Cancer of the breast. If she had been 72, you go for cancer. At 22, you favor benign. But they will not ask you what this is, they will ask what do you do.)

Management: You have to have tissue. Core biopsy is OK, but if negative you don’t stop there. Only excisional biopsy will rule out cancer
A 69 year old lady has a 4 cm hard mass in the right breast, with ill defined borders, movable from the chest wall but not movable within the breast. The skin overlying the mass is retracted an has an “orange peel” appearance…or the nipple became r
Dx: Cancer of the Breast

Diagnostic test: Core or Excisional Bx
A 62 year old lady has an eczematoid lesion in the areola. It has been present for 3 months and it looks to her like “some kind of skin condition” that has not improved or gone away with a variety of lotions and ointments
Dx?
Diagnostic Tes
Dx: Paget’s disease of the breast
(which is a cancer under the areola)

Management: Full thickness punch biopsy of the skin would be OK, but core biopsy or incisional biopsy of the tissue underneath would be OK also
A 42 year old lady hits her breast with a broom handle while doing her housework. She noticed a lump in that area at the time, and one week later the lump is still there. She has a 3 cm hard mass deep inside the affected breast, and some superficial ecch
Dx: Cancer, until proven otherwise

(A classical trap for the unwary. Trauma often brings the area to the attention of the patient...but is not cause of the lump.)
A 58 year old lady discovers a mass in her right axilla. She has a discreet, hard, movable, 2cm mass. Examination of her breast is negative, and she has not enlarged lymph nodes elsewhere
Dx?
Diagnostic Test? (2)
Dx: Cancer, until proven otherwise

(A tough one, but another potential presentation for cancer of the breast. In a younger patient you would think lymphoma. It could still be lymphoma on her.)

Diagnostic test:
1. Mammogram
(we are now looking for an occult primary)
2. Biopsy Node
A 60 year old lady has a routine, screening mammogram. The radiologist reports an irregular area of increased density, with fine microcalcifications, that was not present two year ago on a previous mammogram
Dx?
Further Management?
Dx: Cancer of the Breast

Further management: Stereotactic Radiologically guided Core Biopsy

(If unsatisfactory, the next move would be needle localized excisional biopsy)
A 44 year old lady has a 2cm palpable mass in the upper outer quadrant of her right breast. A core biopsy shows infiltrating ductal carcinoma. The mass is freely movable and her breast is of normal, rather generous size. She has no palpable axillary node
Tx:
1. Segmental Resection (Lumpectomy) and axillary node dissection
2. followed by Radiation Therapy to the remaining breast

Axillary node dissection is to help determine the need for adjuvant systemic therapy
A 62 year old lady has a 4 cm hard mass under the nipple and areola of her rather smallish left breast. A core biopsy has established a diagnosis of infiltrating ductal carcinoma. There are no palpable axillary nodes.

Management?
Management: Modified Radical Mastectomy

(A Lumpectomy is an option only when the tumor is small [in absolute terms and in relation to the breast] and located where most of the breast can be spared.) A modified radical mastectomy is the choice here.

Why go after the axillary nodes when they are not palpable?: Because palpation is notoriously inaccurate in determining the presence or absence of axillary metastasis.
A 44 year old lady shows up in the Emergency Room because she is “bleeding from the breast”. Physical exam shows a huge, fungating, ulcerated mass occupying the entire right breast, and firmly attached to the chest wall. The patient maintains that th
Dx: Advanced Cancer of the Breast

Diagnostic Test: Core or an Incisional biopsy

Management: currently inoperable, and incurable as well...but palliation can be offered.
Chemotherapy is the first line of treatment.
(In many cases the tumor will shrink enough to become operable)
A 37 year old lady has a lumpectomy and axillary dissection for a 3cm infiltrating ductal carcinoma. The pathologist reports clear surgical margins and metastatic cancer in 4 out of 17 axillary nodes.

Management?
Management: Chemotherapy

(Only very small tumors with negative nodes and very favorable histological pattern are “cured” with surgery alone. More extensive tumors need adjuvant systemic therapy, and the rule is that premenopausal women get chemotherapy and postmenopausal women get hormonal therapy.)
A 66 year old lady has a modified radical mastectomy for infiltrating ductal carcinoma of the breast. The pathologist reports that tumor measures 4 cm. in diameter and that 7 out of 22 axillary node are positive for metastasis. The tumor is estrogen and
Management: Hormonal therapy

(The agent used is Tamoxifen)
A 44 year old lady complains bitterly of severe headaches that have been present for several weeks and have not responded to the usual over-the-counter headache remedies. She is two years post-op. from modified radical mastectomy for T3, N2, M0 cancer of
Dx: Brain mets (until proven otherwise)

(Don’t get hung up on the TNM classification, if the numbers are not 1 for the tumor and zero for the nodes and mets, the tumor is bad.)

Diagnostic Test: CT scan of the brain
A 39 year old lady completed her last course of postoperative adjuvant chemotherapy for breast cancer six months ago. She comes to the clinic complaining of constant back pain for about 3 weeks. She is tender to palpation over two well circumscribed area
Dx: Bone mets until proven otherwise

Diagnostic test: Bone Scan
(the most sensitive test for bone mets)

If positive, X-Rays are needed to rule out benign reasons for the scan to “light up”.
A young mother is visiting your office for routine medical care. She happens to have her 18 month old baby with her, and you happen to notice that one of the pupils of the baby is white, while the other one is black.
Dx Differential? (2)
Dx Diff: Retinoblastoma or Cataracts

(An ophthalmological and potentially life-and-death emergency. A white pupil (leukocoria) at this age can be retinoblastoma. This kid needs to see the ophthalmologist not next week, but today or tomorrow. If it turns out to be something more innocent, like a cataract, the kid still needs it corrected to avoid amblyopia.)
Your distant cousins that you have not seen for years visit you and brag about their beautiful baby with “huge, shiny eyes”. They show you a picture that indeed proves their assertion (or the exam booklet will have such a picture).
Dx?
Dx: Huge eyes in babies can be Congenital Glaucoma.

(Tearing will indeed make them shine all the time. If undiagnosed, blindness will ensue.)
A 53 year old lady is in the ER complaining of extremely severe frontal headache. The pain started about one hour ago, shortly after she left the movies where she watched a double feature. On further questioning, she reports seeing halos around the light
Dx: Acute glaucoma

(most are asymptomatic)

Management: An ophthalmologist is needed stat

Tx:
1. Diamox
2. Pilocarpine drops
3. Mannitol
A 32 year old lady presents in the E.R. with swollen, red, hot, tender eyelids on the left eye. She has fever and leukocytosis. When prying the eyelids open, you can ascertain that her pupil is dilated and fixed and that she has very limited motion of th
Dx: Orbital Cellulitis

Management: CT scan

(Ophthalmological emergency that requires immediate consultation)

Tx: Surgical drainage
A frantic mother reaches you on the phone, reporting that her 10 year old boy accidentally splashed Drano on his face and is screaming in pain complaining that his right eye hurts terribly.
Management?
Management: The key is immediate irrigation. Instruct the mother to pry the eye open under the cold water tap at home, and irrigate for about ½ hour before she brings the kid to the hospital.
A 59 year old, myopic gentleman reports “seeing flashes of light” at night, when his eyes are closed. Further questioning reveals that he also sees “floaters” during the day, that they number ten or twenty, and that he also sees a cloud at the to
Dx: Retinal Detachment

(that “cloud” at the top of the visual field is hemorrhage settling at the bottom of the eye)

Management: Another Ophthalmological emergency. The retina specialist will use Laser treatment to “spot weld” the retina back in place
A 77 year old man suddenly loses sight from the right eye. He calls you on the phone 10 minutes after the onset of the problem. He reports no other neurological symptoms.
Dx?
Management?
Dx: Embolic occlusion of the retinal artery

Management: Another ophthalmological emergency...although little can be done for the problem.
He has to get the ER instantly and it might help for him to breathe into a paper bag on route, and have someone press hard on his eye and release repeatedly
A 55 year old man is diagnosed with type two diabetes mellitus. On questioning about eye symptoms he reports that sometimes after a heavy dinner the television becomes blurry and he has to squint to see it clearly.
Dx?
Management?
Dx: Simply DM-related changes in eye

(no big deal: the lens swells and shrinks in response to swings in blood sugar)

Management: regular ophthalmological follow up for retinal complications
A 54 year old obese man gives a history of burning retrosternal pain and “heartburn” that is brought about by bending over, wearing tight clothing or lying flat in bed at night. He gets symptomatic relief from antiacids, but the disease process seems
Dx: Gastroesophageal reflux

Management: Endoscopy and biopsies

(to assess the extent of esophagitis and potential complications before medication)
A 62 year old man describes severe epigastric and substernal pain that he can not characterize well. There is a history suggestive of gastroesophageal reflux, and EKG and cardiac enzymes have been repeatedly negative.
Diagnostic test?
Diagnostic test: Acid Perfusion (Bernstein) test

(it reproduces the pain when the lower esophagus is irrigated with an acid solution to tell if it is only GERD)
A 44 year old black man describes progressive dysphagia that began 3 months ago with difficulty swallowing meat, progressed to soft foods and is now evident for liquids as well. He locates the place where food “sticks” at the lower end of the sternum
Dx: Carcinoma of the Esophagus

Diagnostic test:
1. Barium swallow
2. Endoscopy w/ biopsies
3. CT scan
A 47 year old lady describes difficulty swallowing which she has had for many years. She says that liquids are more difficult to swallow than solids, and she has learned to sit up straight and wait for the fluids to “make it through”. Occasionally sh
Dx: Achalasia

Diagnostic test:
1. Manometry studies (gold standard)
2. CXR w/ barium swallow
3. Endoscopy

Tx:
1. Pneumatic dilation
2. Surgical Myotomy
3. Botox injection (if pt >50 yo is first Tx)
A 24 year old man spends the night cruising bars and drinking heavily. In the wee hours of the morning he is quite drunk and he starts vomiting repeatedly. He initially brings up gastric contents only, but eventually he vomits bright red blood.
Dx?<
Dx: Mallory Weiss tear

Diagnostic test: Endoscopy

(Photocoagulation may be used if needed)
A 24 year old man spends the night cruising bars and drinking heavily. In the wee hours of the morning he is quite drunk and starts vomiting repeatedly. Eventually he has a particularly violent episode of vomiting and he feels a very severe, wrenching ep
Dx: Boerhave’s syndrome

Diagnostic test: Gastrographin swallow

Treatment: Emergency surgical repair

(Prognosis depends on time elapsed between perforation and treatment)
A 55 year old man has an upper G.I. endoscopy done as an outpatient to check on the progress of medical therapy for gastric ulcer. Six hours after the procedure, he returns complaining of severe, constant, retrosternal pain that began shortly after he we
Dx: Instrumental perforation of the esophagus

Diagnostic test: Gastrographin swallow
A 72 year old man has lost 40 pounds of weight over a two or three month period. He gives a history of anorexia for several months, and of vague epigastric discomfort for the past 3 weeks.
Dx?
Diagnostic test?
Dx: Cancer of the stomach

Diagnostic test: Endoscopy and biopsies
A 55 year old patient w/ known PUD presents w/ sudden onset of severe epigastric pain. Physical exam reveals guarding and rebound tenderness.
Dx?
Diagnostic test?
Dx: Anterior Perforated ulcer

Diagnostic test: Chest or Abdominal x-ray to show free air under diaphragm
A 52 year old woman presents due to 3 months of early satiety, weight loss and non-bilious vomiting.
Dx?
Dx: Gastric Outlet Obstruction
A 55 year old patient w/ known PUD presents w/ sudden onset of severe epigastric pain that radiates to the back. Physical exam reveals guarding and rebound tenderness. An Abdominal x-ray does not show free air under diaphragm.
Dx?
Dx: Posterior Perforated ulcer

(An Abdominal x-ray will not show free air under diaphragm if it is a posterior perforation)
A 45 year old Japanese male smoker presents with weight loss and epigastric pain exacerbated by eating.
Dx?
Diagnostic test?
Dx: Gastric Ulcer

Diagnostic test: Endoscopy w/ Bx
A 24 year old patient who was recently a burn victim over 36% of his body presents with epigastric pain exacerbated by eating.
Dx?
Dx: Curling’s Ulcers

(Gastric stress ulcers w/ severe burns. “Burnt paper CURLS”)
A 72 year old recent stroke patient begins to have severe epigastric pain that is exacerbated by eating.
Dx?
Dx: Cushing’s Ulcers

(Gastric ulcer related to severe CNS damage)
A 58 year old woman who is 6 days post-op from a gastrojejunostomy for PUD presents w/ postprandial RUQ pain and nausea. She reports that vomiting relieves her suffering.
Dx?
Diagnostic test?
Tx?
Dx: Afferent Loop syndrome

Diagnostic test: UGI series w/ contrast
(will show afferent loop w/o contrast)

Tx: Endoscopic Balloon dilatation or Surgical revision
(5) causes for an Upper GI Hemorrhage
Mallory’s Vices Gave (her) An Ulcer:

Mallory-Weiss Tear;
Varices;
Gastritis;
AV malformation;
Ulcer
A 54 year old man has had colicky abdominal pain and protracted vomiting for several days. He has developed progressive moderate abdominal distention, and has not had a bowel movement or passed any gas for five days. He has high pitched, loud bowel sound
Dx: Mechanical Intestinal Obstruction, due to adhesions

Management: Nasogastric suction, I.V. fluids and careful observation
A 54 year old man has had colicky abdominal pain and protracted vomiting for several days. He has developed progressive moderate abdominal distention, and has not had a bowel movement or passed any gas for five days. He has high pitched loud bowel sounds
Dx: Strangulated Obstruction
(a loop of bowel is dying –or dead- from compression of the mesenteric blood supply)

Management: Emergency surgery
A 54 year old man has had colicky abdominal pain and protracted vomiting for several days. He has developed progressive moderate abdominal distention, and has not had a bowel movement or passed any gas for five days. He has high-pitched, loud bowel sound
Dx: Mechanical Intestinal Obstruction, due to an incarcerated (potentially strangulated) Hernia.

Management: After suitable fluid replacement needs urgent surgical intervention
A 55 year old lady is being evaluated for protracted diarrhea. On further questioning she gives a bizarre history of episodes of flushing of the face, with expiratory wheezing. A prominent jugular venous pulse is noted on her neck.
Dx?
Diagno
Dx: Carcinoid syndrome.

Diagnostic test:
1. 24 hour Serum determinations of 5-hydroxy-indoleacetic acid (5-HIAA) or 5-HTP;
2. CT scan of abdomen

Tx: Serotonin antagonists;
then...
If Appendiceal < 2cm = Appendectomy.
If Appendiceal > 2cm = Right hemicolectomy;
Small intestinal = resect tumor w/ mesenteric LN
A 22 year old man develops vague periumbilical pain that several hours later becomes sharp, severe, constant and well localized to the right lower quadrant of the abdomen. On physical examination he has abdominal tenderness, guarding and rebound to the r
Dx: Acute Appendicitis

Management: Exploratory laparotomy and appendectomy
A 70 year old male with a history of peripheral vascular disease and hyperlipidemia presents to the ER with diffuse abdominal pain. His BP is 170/100 and his pulse is 90bpm. Supine abdominal radiographs shows air in the wall of the small intestine.
Dx: Small bowel Infarction
A patient presents with pigmented spots on his lips and a history of recurrent colicky abdominal pain.
Dx?
Dx: Peutz-Jeghers syndrome
A 5 year old child presents with increasing irritability, colicky abdominal pain and rectal bleeding w/ stools that have a currant jelly appearance. A mass is palpated in the right lower quadrant.
Dx?
Diagnostic test? (2)
Dx: Intussussception

Diagnostic test: Abdominal x-ray showing air-fluid levels with a stepladder pattern;
Barium enema (which is also therapeutic)
A 59 year old is referred for evaluation because he has been fainting at his job where he operates heavy machinery. He is pale and gaunt, but otherwise his physical exam is remarkable only 4+ occult blood in the stool. Lab studies show a hemogoblin of 5.
Dx: Cancer of the right colon

Diagnostic test: Colonoscopy and biopsies

Treatment: Blood transfusions and eventually Right Hemicolectomy
A 56 year old man has bloody bowel movements. The blood coats the outside of the stool, and has been constipated, and his stools have become of narrow caliber.
Dx?
Diagnostic test?
Dx: Cancer of the distal, left side of the colon

Diagnostic test: Endoscopy and biopsies

(If given choices start with Flexible Sigmoidoscopy)
A 42 year old man has suffered from chronic ulcerative colitis for 20 years. He weights 90 pounds and has had at least 40 hospital admissions for exacerbations of the disease. Due to a recent relapse, he has been placed on high dose steroids and immuran.
Dx: Toxic megacolon

Management: Emergency surgery for the toxic megacolon and removal of the rectum

(but the case illustrates many other indications for surgery: chronic malnutrition, “intractability” and risk of developing cancer)
A 27 year man is recovering from an appendectomy for gangrenous acute appendicitis with perforation and periappendicular abscess. He has been receiving Clindamycin and tobramycin for seven days. Eight hours ago he developed watery diarrhea, crampy abdomi
Dx: Pseudomembranous colitis from overgrowth of Clostridium Difficile

Diagnostic test: Stool cultures (but proctosigmoidoscopy can show a typical picture before the cultures are back)

Management: Stop the clindamycin, give either Vancomycin or Metronidazole, and avoid lomotil
A 60 year old man known to have hemorrhoids reports bright red blood in the toilet paper after evacuation.
Dx?
Management?
Dx: Internal hemorrhoids

Management: Proctosigmoidoscopic Examination
(It is not reassurance and hemorrhoid remedies prescribed by telephone. In all these cases, cancer of the rectum has to be ruled out)
A 60 year old man known to have hemorrhoids complains of anal itching and discomfort, particularly towards the end of the day. He has perianal pain when sitting down and finds himself sitting sideways to avoid the discomfort. He is afebrile.
Dx?
Dx: External hemorrhoids

Management: Proctosigmoidoscopic Examination

(It is not reassurance and hemorrhoid remedies prescribed by telephone. In all these cases, cancer of the rectum has to be ruled out)
A 23 year old lady describes exquisite pain with defecation and blood streaks on the outside of the stools. Because of the pain she avoids having bowel movements and when she finally does, the stools are hard and even more painful. Physical examination c
Dx: Anal Fissure

Management: Exam under Anesthesia

(Even though the clinical picture is classical, cancer still has to be ruled out)

Tx: Lateral Internal Sphincterotomy
A 28 year old male is brought to the office by his mother. Beginning four months ago he has had three operations, done elsewhere, for a perianal fistula, but after each one the area has not healed, but actually the surgical wounds have become bigger. He
Dx: Crohn's Disease

(The perianal area has fantastic blood supply and heals beautifully even though feces bathe the wounds. When it does not, you immediately think of Crohn’s disease)

Diagnostic test: Flexible sigmoidoscopy with Biopsy
(You still have to rule out malignancy)

Top 3 medical Tx:
1. Sulfasalazine
2. Metronidazole
3. Prednisone
A 44 year old man shows up in the E.R. at 11 PM with exquisite perianal pain. He can not sit down, reports that bowel movements are very painful, and that he has been having chills and fever. Physical examination shows a hot, tender, red, fluctuant mass
Dx Ischiorectal abscess

Management: Exam under Anesthesia with Incision and Drainage

(The treatment for all abscesses is drainage. This one is no exception. But as always, cancer has to be ruled out)
A 62 year old man complains of perianal discomfort, and reports that there are streaks of fecal soiling in his underwear. Four months ago he had a perirectal abscess drained surgically. Physical exam shows a perianal opening in the skin, and a cord-liked
Dx: Anal Fistula

First:
Rule-out cancer with Proctosigmoidoscopy

Tx: elective Fistulotomy
A 55-year old, HIV positive man, has a fungating mass growing out of the anus, and rock hard, enlarged lymph nodes on both groins. He has lost a lot of weight, and looks emaciated and ill.
Dx?
Diagnostic Test?
Eventual Tx?
Dx: Squamous cell carcinoma of the anus

Diagnostic test: Biopsies of the fungating mass.

Eventual treatment: Nigro protocol of pre-operative chemotherapy and radiation
A 33 year old man vomits a large amount of bright red blood.
Where can the bleeding be from?
Diagnostic test?
Bleeding from: Tip of the nose to the ligament of Treitz.

Diagnostic test: for all upper G.I. bleeding, start with Endoscopy
A 33 year old man has had three large bowel movements that he describes as made up entirely of dark red blood. The last one was 20 minutes ago. He is diaphoretic, pale, has a blood pressure of 90 over 70 and a pulse rate of 110.
Where is bleeding fr
Bleeding from? Anywhere in GI tract

(The point of the vignette is that something needs to be done to define the area from which he is bleeding. With the available information it could be from anywhere in the G.I. tract)

Management: The first diagnostic move here is to place a Nasogastric tube
A 33 year old man has had three large bowel movements that he describes as made up entirely of dark red blood. The last one was 20 minutes ago. He is diaphoretic, pale, has a blood pressure of 90 over 70 and a pulse rate of 110. A nasogastric tube return
Management: Endoscopy

(Same as if he had been vomiting blood)
A 72 year old man had three large bowel movements that he describes as made up entirely of dark red blood. The last one was two days ago. He is pale, but has normal vital signs. A nasogastric tube returns clear, green fluid without blood.
Diagnostic
Diagnostic test: Upper and Lower Endoscopies

(The clear aspirate is meaningless because he is not bleeding right now. So the guilty territory can be anywhere from the tip of the nose to the anal canal. Across the board, ¾ of all GI bleeding is upper, and virtually all the causes of lower GI bleeding are diseases of the old: diverticulosis, polyps, cancer and angiodysplasias. So, is old, the overall preponderance of upper is balanced by the concentration of lower causes in old people...so it could be anywhere)
A 7 year old boy passes a large bloody bowel movement.
Dx?
Diagnostic test?
Dx: Meckel’s diverticulum
(in this age group)

Diagnostic test: Radioactively labeled Technetium scan
(not the one that tags reds cells, but the one that identifies gastric mucosa)
A 41 year old man has been in the intensive care unit for two weeks, being treated for idiopathic hemorrhagic pancreatitis. He has had several percutaneous drainage procedures for pancreatic abscesses, chest tubes for pleural effusions, and bronchoscopie
Tx: Stress Ulcer

Diagnostic test: Endoscopy

It should have been prevented by keeping the pH of the stomach above 4 with H2 blockers, antiacids or both

Treatment: Angiographic Embolization of the left gastric artery.
A 59 year old man arrives in the E.R. at 2 AM, accompanied by his wife who is wearing curlers on her hair and a robe over her nightgown. He has abdominal pain that began about one hour ago, and is now generalized, constant and extremely severe. He lies m
Dx: Acute Peritonitis (Acute Abdomen)

Management: Emergency Exploratory Laparotomy
A 62 year old man with cirrhosis of the liver and ascites, presents with generalized abdominal pain that started 12 hours ago. He now has moderate tenderness over the entire abdomen, with some guarding and equivocal rebound. He has mild fever and leukocy
Dx: Primary Peritonitis

(Peritonitis in the cirrhotic with ascitis, or the child with nephrosis and ascitis, could be primary peritonitis – which does not need surgery!)

Diagnostic test: Paracentesis with Cultures of the ascitic fluid will yield a single organism

Treatment: Antibiotics
A 43 year old man develops excruciating abdominal pain at 8:18 PM. When seen in the E.R. at 8:50 PM, he has a rigid abdomen, lies motionless in the examining table, has no bowel sounds and is obviously in great pain, which he describes as constant. X-Ray
Dx: Acute abdomen plus perforated GI tract
(perforated duodenal ulcer in most cases)

Management: Emergency exploratory laparotomy
A 44 year old alcoholic male presents with severe epigastric pain that began shortly after a heavy bout of alcoholic intake, and reached maximum intensity over a period of two hours. The pain is constant, radiates straight through to the back and is acco
Dx: Acute pancreatitis

Diagnostic test: Serum and Urinary Amylase and Lipase

If unclear: CT scan
(or in a day or two if there is no improvement)

Management: NPO, NG suction, IV fluids.
A 43 year old obese lady, mother of six children, has severe right upper quadrant abdominal pain that began six hours ago. The pain was colicky at first, radiated to the right shoulder and around towards the back, and was accompanied by nausea and vomiti
Dx: Acute cholecystitis

Diagnostic test: Ultrasound

(If equivocal, an “HIDA” scan: radionuclide excretion scan)

Management: “cool down” the process

Surgery will follow
A 52 year old man has right flank colicky pain of sudden onset that radiates to the inner thigh and scrotum. There is microscopic hematuria.
Dx?
Diagnostic test? (2)
Dx: Ureteral colic

Diagnostic test: Urological evaluation always begins with a Plain Film of the abdomen (a “KUB”)

Ultrasound often is the next step
(but traditionally it has been intravenous pyelogram)
A 59 year old lady has a history of three prior episodes of left lower quadrant abdominal pain for which she was briefly hospitalized and treated with antibiotics. Now she has left lower quadrant pain, tenderness, and a vaguely palpable mass. She has fev
Dx: Acute diverticulitis

Diagnostic test: CT scan
(Colonoscopy is not safe in acute setting)

Management: Elective Sigmoid resection
(for recurrent attacks, like this case or if she does not respond to medical Tx from initial attack or gets worse)

(Treatment is medical for the acute attack: antibiotics, NPO)
An 82 year old man develops severe abdominal distension, nausea, vomiting and colicky abdominal pain. He has not passed any gas or stool for the past 12 hours. He has a tympanitic abdomen with hyperactive bowel sounds. X-Ray shows distended loops of smal
Dx: Volvulus of the sigmoid

Management: Proctosigmoidoscopy should relieve the obstruction

(Rectal tube is another option. Eventually surgery to prevent recurrences could be considered)
A 79 year old man with atrial fibrillation develops and acute abdomen. He has a silent abdomen, with diffuse tenderness and mild rebound. There is a trace of blood in the rectal exam. He has acidosis and looks quite sick. X-Rays show distended small bowe
Dx: Emboli of Mesenteric vessels

(Acute abdomen present in the elderly who has atrial fibrillation, brings to mind embolic occlusion of the mesenteric vessels. Acidosis frequently ensues, and blood in the stool is often seen)

Mild Tx: Observe only
Moderate Tx (fever and inc WBC only): IV Antibiotics

Severe Tx (Peritoneal signs): Exploratory Lap with Colostomy
A 53 year old man with cirrhosis of the liver develops malaise, vague right upper quadrant abdominal discomfort and 20 pound weight loss. Physical exam shows a palpable mass that seems to arise from the left lobe of the liver. Alpha feto protein is signi
Dx: Liver cell carcinoma

Diagnostic test: CT scan

Tx: If confined to one lobe, Resection.
A 53 year old man develops vague right upper quadrant abdominal discomfort and a 20 pound weight loss. Physical exam shows a palpable liver with nodularity. Two years ago he had a right hemicolectomy for cancer of the ascending colon. His carcinoembryoge
Dx: Metastasis to the liver from colon cancer

Diagnostic test: CT scan

Tx: If mets are confined to one lobe: Resection.

(Otherwise, Chemotherapy if he has not had it)
A 24 year old lady develops moderate, generalized abdominal pain of sudden onset, and shortly thereafter faints. At the time of evaluation in the ER she is pale, tachycardic, and hypotensive. The abdomen is mildly distended and tender, and she has a hemo
Dx: Bleeding from a ruptured Hepatic Adenoma, secondary to birth control pills.

Management:
CAT scan
(will confirm bleeding and probably show the liver adenoma as well)

Tx: Surgery
A 44 year old lady is recovering from an episode of acute ascending cholangitis secondary to choledocholithiasis. She develops fever and leukocytosis and some tenderness in the right upper quadrant. An ultrasound reveals a liver mass.
Dx?
Manag
Dx: Pyogenic abscess

Management: it needs to be drained (the radiologists will do it percutaneously)
A 29 year old migrant worker from Mexico develops fever and leukocytosis, as well as tenderness over the liver when the area is percussed. He has mild jaundice and an elevated alkaline phosphatase. Ultrasound of the right upper abdominal area shows a nor
Dx: Amebic abscess
(very common in Mexico)

Management: Serology for Amebic titers and start on Metronidazole
(This one Abscess that does not have to be drained. Get serology for amebic titers, and start the patient on Metranidazole. Prompt improvement will tell you that you are on the right track...serologies in 3 weeks will confirm. Don’t fall for an option that suggests aspirating the pus and sending it for culture, you can not grow the ameba from the pus)
A 42 year old lady is jaundiced. She has a total bilirubin of 6 and the laboratory reports that the unconjugated, indirect bilirubin is 6 and the direct, conjugated bilirubin is zero. She has no bile in the urine.
Dx?
Management?
Dx: Hemolytic Jaundice

Management: Try to figure out what is chewing her red cells.
A 19 year old college student returns from a trip to Cancun, and two weeks later develops malaise, weakness and anorexia. A week later he notices jaundice. When he presents for evaluation his total bilirubin is 12, with 7 indirect and 5 direct. His alkal
Dx: Hepatocellular jaundice

Management: Get serologies to confirm diagnosis and type of Hepatitis
A patient with progressive jaundice which has been present for four weeks is found to have a total bilirubin of 22, with 16 direct and 6 indirect, and minimally elevated SGOT. The alkaline phosphatase was twice normal value couple of weeks ago, and now i
Dx: Obstructive jaundice

Diagnostic test? Ultrasound

(looking for dilated intrahepatic ducts, possibly dilated extrahepatic ducts as well, and if we get lucky a finding of gallstones)
A 40 year old, obese mother of five children presents with progressive jaundice which she first noticed four weeks ago. She has a total bilirubin of 22, with 16 direct and 6 indirect, and minimally elevated SGOT. The alkaline phosphatase is about six tim
Dx: Obstructive jaundice

Diagnostic test: Ultrasound
(If you need more tests after that, ERCP is the next move, which could also be used to remove the stones from the common duct)

Tx: Cholecystectomy
A 66 year old man presents with progressive jaundice which he first noticed six week ago. He has a total bilirubin of 22, with 16 direct and 6 indirect, and minimally elevated SGOT. The alkaline phosphatase is about six times the upper limit of normal. H
Dx: Malignant obstructive jaundice.
(“Silent” obstructive jaundice is more likely to be due to tumor. A distended gallbladder is an ominous sign: when stones are the source of the problem, the gallbladder is thick-walled, non-pliable)

Management: CAT scan and ERCP
A 66 year old man presents with progressive jaundice which he first noticed six weeks ago. He has a total bilirubin of 22, with 16 direct and 6 indirect, and minimally elevated SGOT. The alkaline phosphatase is about six times the upper limit of normal.
Dx: Malignant, but lucky... probably Cholangiocarcinoma at the lower end of the common duct.

Next step: get brushings of the common duct for cytological diagnosis.

Tx: He could be cured with a pancreatoduodenectomy
(Whipple operation)
A 64 year old lady presents with progressive jaundice which she first noticed two weeks ago. She has a total bilirubin of 12, with 8 direct and 4 indirect, and minimally elevated SGOT. The alkaline phosphatase is about ten times the upper limit of normal
Dx: Malignant Obstructive jaundice
(The coincidence of slowly bleeding into the GI tract at the same time that she develops obstructive jaundice points to an Ampullary carcinoma, another malignancy that can be cured with Radical surgery)

Diagnostic test: Endoscopy
A 56 year old man presents with progressive jaundice which he first noticed six weeks ago. He has a total bilirubin of 22, with 16 direct and 6 indirect, and minimally elevated SGOT. He alkaline phosphatase is about eight times the upper limit of normal.
Dx: Cancer of the head of the pancreas
(Terrible prognosis)

Diagnostic test: CAT scan –which may show the mass in the head of the pancreas;
then ERCP –which will probably show obstruction of both common duct and pancreatic duct
A white, fat, female, aged 40 and mother of five children gives a history of repeated episodes of right upper quadrant abdominal pain brought about by the ingestion of fatty foods, and relieved by the administration of anticholinergic medications. The pa
Dx: Gallstones, with biliary colic

Diagnostic test: Ultrasound

Tx: Elective cholecystectomy
A 43 year old obese lady, mother of six children, has severe right upper quadrant abdominal pain that began three days ago. The pain was colicky at first, but has been constant for the past two and a half days. She has tenderness to deep palpation, muscl
Dx: Acute Ascending Cholangitis

Further test:
Ultrasound might confirm dilated ducts.

Management:
Emergency decompression of the biliary tract...
ERCP is the first choice, but PTC
(percutaneous transhepatic cholangiogram) is another option
A white, fat, female, aged 40 and mother of five children gives a history of repeated episodes of right upper quadrant abdominal pain brought about by the ingestions of fatty foods, and relieved by the administration of anticholinergic medications. The p
Dx: She passed a common duct stone and had a transient episode of Cholangitis (the shaking chill, the high phosphatase) and a bit of Biliary Pancreatitis (the high amylase).

Diagnostic test: Ultrasound (It will confirm the diagnosis of gallstones)

Management: If she continues to get well, elective Cholecystectomy.
If she deteriorates, she may have the stone still impacted at the Ampulla of Vater, and may need ERCP and sphincterotomy to extract it
A 33 year old, alcoholic male, shows up in the E.R. with epigastric and mid-abdominal pain that began 12 hours ago shortly after the ingestion of a large meal. The pain is constant, very severe, and it radiates straight through to the back. He vomited tw
Dx: Acute edematous pancreatitis.

Management: put the pancreas at rest...NPO, NG suction, IV fluids
A 56 year old alcoholic male is admitted with a clinical picture of acute upper abdominal pain. The pain is constant, radiates straight through to the back, and is extremely severe. He has a serum amylase of 800, WBC of 18,000 blood glucose of 150, serum
Dx: Hemorrhagic Pancreatitis
(In fact, he is in deep trouble, with at least eight of Ranson’s criteria predicting 80 to 100% mortality)

Management/test: Very intensive support will be needed, but the common pathway to death from complication of hemorrhagic pancreatitis frequently is by way of pancreatic abscesses that need to be drained as soon as they appear. Thus serial CT scans will be required.
A 57 year old alcoholic male is being treated for acute hemorrhagic pancreatis. He was in the intensive care unit for one week, required chest tubes for pleural effusion, and was on a respirator for several days, but eventually improved enough to be tran
Dx: Pancreatic abscess

Diagnostic test: CT scan

Tx: Drainage
A 49 year old alcoholic male presents with ill-defined upper abdominal discomfort and early satiety. On physical exam he has a large epigastric mass that is deep within the abdomen, and actually hard to define. He was discharged from the hospital 5 weeks
Dx: Pancreatic pseudocyst

Diagnostic test: You could diagnose it on the cheap with an ultrasound, but CT scan is probably the best choice.

Tx: It will need to be drained, and the radiologist will do it with CT guidance
A 55 year old lady presents with vague upper abdominal discomfort, early satiety and a large but ill-defined epigastric mass. Five weeks ago she was involved in an automobile accident where she hit the upper abdomen against the steering wheel.
Dx?
Dx: Pancreatic pseudocyst, secondary to trauma

Diagnostic test: CT scan
A disheveled, malnourished individual shows up in the emergency room requesting medication for pain. He smells of alcohol and complains bitterly of constant epigastric pain, radiating straight through to the back that he says he has had for several years
Dx: Chronic pancreatitis

Diagnostic test: AXR visualizing calcifications

Management: Stop alcohol, replacement of pancreatic enzymes and control of the diabetes; ERCP
On the first post-operative day after an open cholecystectomy, a patient has a temperature of 101.
Dx?
Diagnostic test?
Management? (2 together)
Dx: Atelectasis

Diagnostic test: Chest X-ray

Management:
1. Incentive Spirometry
2. Encourage deep breathing and coughing
On the third post-operative day after an open cholecystectomy, a patient develops a temperature of 101.
Dx?
Diagnostic test?
Tx?
Urinary tract infection

Diagnostic Test: Urinalysis and Urinary culture

Tx: appropriate Antibiotics
On the fourth post-operative day after an open cholecystectomy, a patient develops a temperature of 101. There is tenderness to deep palpation in the calf, particularly when the foot is dorsiflexed.
Dx?
Diagnostic test?
Tx?
Dx: Deep Venous Thrombosis

Diagnostic test: Duplex ultrasound
(Doppler flow plus real time B-mode)

Tx: Anticoagulation to prevent thrombus propagation
Seven days after an inguinal hernia repair, a patient returns to the clinic because of fever. The wound is red, hot and tender.
Dx?
Management? (3 steps)
Wound infection

Management:
1. Open the wound
2. Drain the pus
3. Pack it and leave it open
Two weeks after an open cholecystectomy a patient develops fever and leukocytosis. The wound is healing well and does not appear to be infected.
Dx?
Where is greatest possibility? (2)
Diagnostic test?
Tx?
Dx: Deep Abscess

Places: Subphrenic or Subhepatic
(Had the operation been an appendectomy, pelvic abscess would be the first pick)

Diagnostic test: CT scan to find the abscess and to guide the radiologist for the (Tx) Percutaneous Drainage.
On the fifth post-operative day after a right hemicolectomy for cancer, the dressings covering the midline abdominal incision are found to be soaked with a clear, pinkish, salmon-colored fluid.
Dx?
Management? (3 steps)
Dx: Wound dehiscence

Management:
1. Keep the patient in bed
2. Tape his belly together
3. Schedule surgery for re-closure of the wound if the patient can take the re-operation.
(If too sick, the development of a ventral incisional hernia may have to be accepted now and repaired later)
Following the discovery of the copious, salmon colored, pinkish clear fluid along the post-op abdominal incision, the patient gets out of bed, or sneezes forcefully, and you are confronted with a bucket-full of small bowel
Dx?
Management? (2 st
Dx: Evisceration

Management:
1. keep the bowel covered and moist with sterile dressings
2. Rush the patient to the OR for re-closure
A 62 year old lady was drinking her morning cup of coffee at the same time she was applying her makeup, and she noticed in the mirror that there was a lump in the lower part of her neck, visible when she swallowed. She consult you for this and on physica
Diagnostic test: FNA
A 21 year old college student is found on a routine physical examination to have a single, 2cm nodule in the thyroid gland. The young man had radiation to his head and neck when he was thirteen years old because of persistent acne. His thyroid function t
Diagnostic test: FNA

Tx: Surgical removal
(due to radiation leading to cancer)
A 44 year old lady has a palpable mass in her thyroid gland. She also describes losing weight in spite of a ravenous appetite, palpitations and heat intolerance. She is a thin lady, fidgety and constantly moving, with moist skin and a pulse rate of 105.<
Dx: A “hot” Adenoma

Management/test:
1. confirm hyperthyroidism by measuring Free T4
2. Confirm source of the excessive hormone with Radioactive Iodine Scan
3. give Beta-blocker

Tx: Surgery
(after Beta blocking)
A 22 year old male has a 2 cm round firm mass in the lateral aspect of his neck, which has been present for four months. Clinically this is assumed to be an enlarged jugular lymph node and it is eventually removed surgically. The pathologist reports that
Dx: Follicular Carcinoma of the Thyroid (metastitic)
(There is no such thing as “lateral aberrant thyroid”)

Diagnostic test: Look for the primary with a Thyroid Scan.

Tx: Eventually Surgery
An automated blood chemistry panel done during the course of a routine medical examination indicates that an asymptomatic patient has a serum calcium of 12.1 in a lab where the upper limit of normal is 9.5. Repeated determinations are consistently betwee
Dx: Parathyroid Adenoma

Diagnostic test: PTH determination and Sistimibi scan to localize the adenoma

Tx: Surgical excision
A 32 year old woman is admitted to the psychiatry unit because of wild mood swings. She is found to be hypertensive and diabetic and to have osteoporosis. (she had not been aware of such diagnosis beforehand). It is also ascertained that she has been ame
Dx: Cushings Dz
(The appearance is so typical, that you will probably be given a photograph on the test, with an accompanying brief vignette)

Diagnostic test:
1. AM and PM cortisol determinations
2. Dexamethasone suppression tests
3. MRI of the head looking for the pituitary microadenoma

Tx: removed by the trans-nasal, trans-sphenoidal route
A 28 year old lady has virulent peptic ulcer disease. Extensive medical management including eradication of H.Pylori fails to heal her ulcers. She has several duodenal ulcers in the first and second portions of the duodenum. She has watery diarrhea.
Dx: Gastrinoma (Zollinger-Ellison)

Diagnostic test:
1. Serum gastrin
2. CT scans (or MRI) of the pancreas looking for the tumor

Tx: Surgical excision
A second year medical student is hospitalized for a neurological work-up for a seizure disorder of recent onset. During one of his convulsions it is determined that his blood sugar is extremely low. Further work-up shows that he has high levels of insuli
Dx: Exogenous administration of insulin
(If the C-peptide had been high along with the insulin level, the diagnosis would have been insulinoma)

Management: Psychiatric evaluation and counseling
(He is faking the disease. If it had been insulinoma, CT scan or MRI looking for the tumor in the pancreas, to be subsequently removed surgically)
A 48 year old lady has had severe, migratory necrolytic dermatitis for several years, unresponsive to all kinds of “herbs and unguents”. She is thin, has mild stomatitis and mild diabetes mellitus.
Dx?
Diagnostic test? (2)
Tx?
If
Dx: Glucagonoma

Diagnostic test:
1. Determine Glucagon levels
2. CT scan or MRI looking for the tumor in the pancreas.

Tx: Surgery will follow

If inoperable:
1. Somatostatin can help symptomatically
2. Streptozocin is the indicated chemotherapeutic agent
A 45 year old lady comes to your office for a “regular checkup”. On repeated determinations you confirm the fact that she is hypertensive. When she was in your office three years ago, her blood pressure was normal. Laboratory studies at this time sho
Dx: Hyperaldosteronism or Adrenal Adenoma

Diagnostic test:
1. Aldosterone and renin levels.
2. If confirmatory (aldo high, renin low) proceed with determinations lying down and sitting up, to differentiate Hyperplasia (not surgical) from Adenoma (surgical).

Hyperplasia Tx: Aldactone

Adenoma Tx: Imaging studies (CT scan or MRI) and Surgery
A thin, hyperactive 38 year old lady is frustrated by the inability of her physicians to help her. She has episodes of severe pounding headache, with palpitations, profuse perspiration and pallor, but by the time she gets to her doctor’s office she che
Dx: Pheochromocytoma

Diagnostic test:
1. 24hr urinary determination of metanephrine and VMA (Vanillylmandelic acid)
2. CT scan of adrenal glands

Meds before surgery: Alpha-blockers
A 17 year old man is found to have a blood pressure of 190/115. This is checked repeatedly in both arms and it is always found to be elevated, but when checked in the legs it is found to be normal.
Dx?
Diagnostic test? (2 steps)
Tx?
Dx: Coarctation of the Aorta

Diagnostic test:
1. Chest X-Ray, looking for scalloping of the ribs
2. Aortogram

Tx: Surgery
A 23 year old lady has had severe hypertension for two years, and she does not respond well to the usual medical treatment for that condition. A bruit can be faintly heard over her upper abdomen.
Dx?
Diagnostic test?
Tx? (2 possible)
Dx: Renovascular Hypertension due to Fibromuscular Dysplasia

Diagnostic test: Arteriogram will precede (Tx) Surgical correction or Balloon dilatation
A 72 year old man with multiple manifestations of arteriosclerotic occlusive disease has hypertension of relatively recent onset, and is refractory to the usual medical therapy. He has a faint bruit over the upper abdomen.
Dx?
Dx: Renovascular Hypertension due to arteriosclerotic plaque at the origin of the Renal Artery⬦or arteries
(this is usually bilateral)
Within eight hours after birth, it is noted that a baby has excessive salivation. A small, soft nasogastric tube is inserted and the baby is taken to X-Ray to have a “babygram” done. The film shows the tube coiled back upon itself in the upper chest.
Dx: Tracheo-esophageal fistula
(the most common type with proximal blind esophageal pouch and distal TE fistula)

Management:
1. Rule-out the associated anomalies (“VACTER”: vertebral, anal, cardiac, TE and renal/radial). The vertebral and radial will be seen in the same X-ray you already took, you need Echo for the heart, Sonogram for the kidneys and Physical Exam for the anus.

Tx: Surgical repair
A newborn baby is found on physical exam to have an imperforate anus.

Management? (2 steps)
Management:
1. This is part of the “VACTER” (vertebral, anal, cardiac, TE and renal/radial) group, so look for the others as mentioned.
2. For the imperforate anus, look for a fistula nearby (to the vagina in little girls, to the perineum in little boys), which will help determine the level of the blind pouch and the timing and type of surgery (primary repair versus colostomy and repair later).
A newborn baby is noted to be tachypneic, cyanotic and grunting. The abdomen is scaphoid and there are bowel sounds heard over the left chest. An X-Ray confirms that there is bowel in the left thorax. Shortly thereafter, the baby develops significant hyp
Dx: Congenital Diaphragmatic Hernia

Management:
1. keep the kid alive with endotracheal intubation
2. Hyperventilation (careful not to blow up the other lung)
3. Sedation
4. NG suction
(Tx: The main problem is the hypoplastic lung. It is better to wait 36 to 48 hours to do Surgery to allow transition from fetal circulation to newborn circulation)
At the time of birth it is noted that a child has a large abdominal wall defect to the right of the umbilicus. There is a normal cord, but protruding from the defect there is a matted mass of angry looking, edematous bowel loops.
Dx?
Tx?
Dx: Gastroschisis

Tx: Pediatric Surgeon must get the bowel back into the belly; they may need to use a silicon “silo” to gradually close the abdominal wall defect.
A newborn baby is noted to have a shiny, thin, membranous sac at the base of the umbilical cord. Inside the sac one can see part of the liver, and loops of normal looking bowel.
Dx?
Management?
Tx?
Dx: Omphalocele

Management: Look for other congenital defects. These kids can have a host of other congenital defects
Tx: Repair is performed by a Pediatric surgeon
A newborn is noted to have a moist medallion of mucosae occupying the lower abdominal wall, above the pubis and below the umbilicus. It is clear that urine is constantly bathing this congential anomaly.
Dx?
what is important regarding this repa
Dx: Exstrophy of the urinary bladder

Important: Repair must be done within the first 48 hours, or it will not have a good chance to succeed. It takes time to arrange for transfer of a newborn baby to a distant city that specializes in this repair. If a day or two are wasted before arrangements are made, it will be too late
Half an hour after the first feed, a baby vomits greenish fluid. The mother had polyhydramnios and the baby has Down’s syndrome. X-Ray shows a “double bubble sign”: a large air fluid level in the stomach, and smaller one in the first portion of the
Dx: Duodenal Atresia or Annular Pancreas
(innocent vomit is clear-whitish. Green vomiting in the newborn is bad news. It means something serious)

Management: Look for other congenital anomalies first

Tx: Emergency Surgery
Half an hour after the first feed, a baby vomits greenish fluid. X-Ray shows a "double bubble sign⬝: a large air fluid level in the stomach, and a smaller one in the first portion of the duodenum. There is air in the distal bowel, beyond the duode
Dx:
1. Incomplete obstruction from duodenal stenosis,
2. Annular Pancreas,
3. Malrotation of bowel

Diagnostic test: Contrast enema
(and if not diagnostic order a water-soluble gastrographin Upper GI study)
A newborn baby has repeated green vomiting during the first day of life, and does not pass any meconium. Except for abdominal distention, the baby is otherwise normal. X-Ray shows multiple air fluid levels and distended loops of bowel.
Dx?
Caus
Dx: Intestinal atresia

Cause: Vascular accident in utero
(thus there are no other congenital anomalies to look for, but there may be multiple points of atresia)
A very premature baby develops feeding intolerance, abdominal distention and a rapidly dropping platelet count. The baby is four days old, and was treated with indomethacin for a patent ductus.
Dx?
Management? (3 together)
Reasons for surg
Dx: Necrotizing Enterocolitis

Management:
1. Stop all feedings
2. Broad spectrum antibiotics
3. IV fluids/nutrition

Tx: Surgical intervention if they develop abdominal wall erythema, air in the biliary tree or pneumoperitoneum
A three day old, full term baby is brought in because of feeding intolerance and bilious vomiting. X-Ray shows multiple dilated loops of small bowel and a “ground glass” appearance in the lower abdomen. The mother has cystic fibrosis.
Dx?
M
Dx: Meconium Ileus

Management:
1. Gastrografin enema may be both diagnostic and therapeutic, so it is the obvious first choice.
2. If unsuccessful, surgery may be needed.
3. The kid has cystic fibrosis, and management of the other manifestations of the disease will also be needed
A three week old baby has had “trouble feeding” and it is not quite growing well. He now has bilious vomiting and is brought in for evaluation. X-Ray shows a classical “double bubble”, along with normal looking gas pattern in the rest of the bowe
Dx: Malrotation of the bowel
(not all will show up on day one)

Diagnostic test: Contrast enema to verify the malrotation

Tx: Emergency surgery
A 3 week old first-born, full term baby boy began to vomit three days ago. The vomiting is projectile, has no bile in it, follows each feeding and the baby is hungry and eager to eat again after he vomits. He looks somewhat dehydrated and has visible gas
Dx: Hypertrophic Pyloric Stenosis

Management:
1. Check electrolytes: hypokalemic, hypochloremic metabolic alkalosis may have developed (correct it).
2. Rehydrate

Tx: Ramsted Pyloromyotomy
An 8 week old baby is brought in because of persistent, progressively increasing jaundice. The bilirubin is significantly elevated and about two thirds of it is conjugated, direct bilirubin. Ultrasound rules out extrahepatic masses, serology is negative
Dx: Biliary Atresia

Diagnostic test:
1. HIDA scan
2. Percutaneous Liver Biopsy

Tx: Exploratory laparotomy
A two month old baby boy is brought in because of chronic constipation. The kid has abdominal distention, and plain X-Rays show gas in dilated loops of bowel throughout the abdomen. Rectal exam is followed by expulsion of stool and flatus, with remarkabl
Dx: Hirschsprungs’ disease (aganglionic megacolon)

Diagnostic test:
1. Barium enema will define the normal-looking aganglionic distal colon and the abnormal-looking thickness
2. Biopsy of the rectal mucosa

Tx: Surgical excision of aganglionic segment
A 9 month old, chubby, healthy looking little boy has episodes of colicky abdominal pain that make him double up and squat. The pain lasts for about one minute, and the kid looks perfectly happy and normal until he gets another colick. Physical exam show
Dx: Intussusception

Management: Barium enema is both diagnostic and therapeutic in most cases.

Tx: If reduction is not achieved radiologically, exploratory laparotomy and manual reduction will be needed
A one year old baby is referred to the University Hospital for treatment of a subdural hematoma. In the admission examination it is noted that the baby has retinal hemorrhages.
Dx?
Child Abuse
A one year old child is brought in with second degree burns of both buttocks. The stepfather relates that the child fell into a hot tub.
Dx?
Child Abuse
A three year old girl is brought in for treatment of a fractured humerus. The mother relates that the girl fell from her crib. X-Rays show evidence of other older fractures at various stages of healing in different bones.
Dx?
Child Abuse
A 4 year old boy passes a large bloody bowel movement.
Dx?
Diagnostic test?
Tx?
Dx: Meckel’s diverticulum

Diagnostic test: Radioisotope scan looking for gastric mucosa in the lower abdomen

Tx: Surgical excision
A 15 year old girl has a round, 1cm cystic mass in the midline of her neck at the level of the hyoid bone. When the mass is palpated at the same time that the tongue is pulled, there seems to be a connection between the two. The mass has been present for
Dx: Thyroglossal Duct Cyst

Tx: Sistrunk operation
(removal of the mass and the track to the base of the tongue, along with the medial segment of the hyoid bone).
An 18 year old woman has a 4cm fluctuant round mass on the side of her neck, just beneath and in front of the sternomastoid. She reports that is has been there at least 10 years, although she thinks that it has become somewhat larger in the last year or
Dx: Branchial Cleft Cyst

Tx: Elective surgical removal
A 6 year old child has a mushy, fluid filled mass at the base of the neck, that has been noted for several years. The mass is about 6 cm. in diameter, occupies most of the supraclavicular area and seems by physical exam to go deeper into the neck and che
Dx: Cystic hygroma

Diagnostic test: CT scan to see how deep this thing goes.
(They can extend down into the chest and mediastinum)

Tx: Surgical removal will eventually be done
A 22 year old lady notices an enlarged lymph node in her neck. The node is in the jugular chain, measures about 1.5cm, is not tender, and was discovered by the patient yesterday. The rest of the history and physical exam are unremarkable.
Management
Management: Reschedule an appointment for 3 weeks to see its progress

(If the node has gone away by then, it was inflammatory and nothing further is needed. If it’s still there, it could be neoplastic and something needs to be done)
A 22 year old lady seeks help regarding an enlarged lymph node in her neck. The node is in the jugular chain, measures about 2cm, is firm, not tender, and was discovered by the patient six weeks ago. There is a history of low grade fever and night sweats
Dx: Lymphoma (most likely)

Disgnostic test: Tissue diagnosis will be needed. You can start with FNA of the available nodes, but eventual node biopsy will be needed to establish not only the diagnosis but also the type of lymphoma
A 72 year old man has 4cm hard mass in the left supraclavicular area. The mass is movable, non tender and has been present for three months. The patient has had a 20 pound weight loss in the past two months, but is otherwise asymptomatic.
Dx?
M
Dx: Malignant mets to a supraclavicular node from a primary tumor below the neck.

Management:
1. Look for the obvious primary tumors: lung, stomach, colon, pancreas, and kidney
2. The node itself will eventually be Biopsied
A 69 year old man who smokes and drinks and has rotten teeth has a hard, fixed, 4cm mass in his neck. The mass is just medial and in front of the sternomastoid muscle, at the level of the upper notch of the Thyroid cartilage. It has been there for at lea
Dx: Metastatic squamous cell carcinoma to a jugular chain node, from a primary in the mucosa of the head and neck
(oro-pharyngeal-laryngeal territory)

Diagnostic test: Triple Endoscopy
(examination under anesthesia of the mouth, pharynx, larynx, esophagus and tracheobronchial tree)

(Don’t biopsy the node! FNA is OK if Triple endoscopy not available)
A 69 year old man who smokes and drinks and has rotten teeth has hoarseness that has persisted for six weeks in spite of antibiotic therapy
Dx?
Diagnostic test?
Dx: Squamous cell carcinoma of the mucosa of the head and neck

Diagnostic test: Triple endoscopy to find and biopsy the primary tumor
A 69 year old man who smokes and drinks and has rotten teeth has a painless ulcer in the floor of the mouth that has been present for 6 weeks and has not healed.
Dx?
Diagnostic test?
Squamous cell carcinoma of the mucosa of the head and neck

Diagnostic test: Triple endoscopy to find and biopsy the primary tumor
A 69 year old man who smokes and drinks and has rotten teeth has unilateral ear ache that has not gone away in 6 weeks. Physical examination shows serious otitis media on that side, but not on the other.
Dx?
Diagnostic test?
Dx: Squamous cell carcinoma of the mucosa of the head and neck

Diagnostic test: Triple endoscopy to find and biopsy the primary tumor
A 52 year old man complains of hearing loss. When tested he is found to have unilateral sensory hearing loss on one side only. He hoes not engage in any activity (such as sport shooting) that would subject that ear to noise that spares the other side.
Dx: Acoustic Nerve Neuroma
(Unilateral versions of common ENT problems in the adult suggest malignancy. Note that if the hearing loss had been conductive, a Cerumen Plug would be the obvious first diagnosis)

Diagnostic test: MRI looking for the tumor
A 56 year old man develops slow, progressive paralysis of the facial nerve on one side. It took several weeks for the full blown paralysis to become obvious, and it has been present now for three months. It affects both the forehead as well as the lower
Dx: Gradual, unilateral nerve paralysis suggests a neoplastic process

Diagnostic test: Gadolinium enhanced MRI
A 45 year old man presents with a 2cm firm mass in front of the left ear, which has been present for four months. The mass is deep to the skin and it is painless. The patient has normal function of the facial nerve.
Dx?
Management?
Dx: Pleomorphic adenoma (mixed tumor) of the parotid gland

Management: Referral to a head and neck surgeon for formal superficial parotidectomy
(FNA is appropriate, but the point of the question will be to bring out the fact that parotid masses are never biopsied in the office or under local anesthesia)
A 65 year old man present with a 4cm hard mass in front of the left ear, which has been present for six months. The mass is deep to the skin and it is fixed. He has constant pain in the area, and for the past two months has had gradual progression of lef
Dx: Cancer of the parotid gland

Management: Referral to a head and neck surgeon for formal superficial parotidectomy
(Amateurs should not mess with parotid)
A two year old boy has unilateral ear ache.
Dx?
Dx: Unilateral versions of common bilateral ENT conditions in toddlers suggest Foreign Body
A two year old has unilateral foul smelling purulent rhinorrhea.
Dx?
Dx: Unilateral versions of common bilateral ENT conditions in toddlers suggest foreign body
A two year old has unilateral wheezing and the lung on that side looks darker on X-Rays (more air) than the other side.
Dx?
Dx: Unilateral versions of common bilateral ENT conditions in toddlers suggest foreign body
A 4 year old child is brought by his mother to the emergency room because “she is sure that he must have swallowed a marble”. The kid was indeed playing with marbles and apparently completely healthy when he was put to bed, but four hours later he ha
Dx: Acute Epiglotitis

Diagnostic test: Lateral X-ray of the neck

Management: A real emergency where expert help is needed!
1. Ready to use bag and mask if needed.
2. OR for Nasotracheal Intubation.
3. Start IV antibiotics along the way for H.Pylori

Bradychardia develops: Atropine will help, but hypoxia is the problem.
A 45 year old lady with a history of a recent tooth infection shows up with a huge, hot, red, tender, fluctuant mass occupying the left lower side of her face and upper neck, including the underside of the mouth. The mass pushes up the floor of the mouth
Dx: Ludwigs’ Angina
(An abscess of the floor of the mouth)

Tx:
1. Tracheostomy
2. Incision & Drainage of the abscess
A 29 year old lady calls your office at 10 AM with the history that she woke up that morning with one side of her face paralyzed.
Dx?
Management?
Dx: Bell’s palsy

Management: Immediate anti-viral medication
(the process is idiopathic and will resolve spontaneously in most cases)
A patient with multiple trauma from a car accident is being attended to in the emergency room. As multiple invasive things are done to him, he repeatedly grimaces with pain. The next day it is noted that he has a facial nerve paralysis on one side.
Dx: Paralysis from Edema

(Trauma to the temporal bone can certainly transect the facial nerve, but when that happens the nerve is paralyzed right there and then. Nothing needs to be done...it will correct itself)
Your office receives a phone call from Mrs. Rodriguez. You know this middle aged lady very well because you have repeatedly treated her in the past for episodes of sinusitis. In fact, six days ago you started her on decongestants and oral antibiotics for
Dx: Cavernous Sinus Thrombosis or Orbital Cellulitis

Management: This is a real emergency (fact that is most likely questioned).
1. Immediate Hospitalization,
2. high dose IV Antibiotic treatment
3. Surgical Drainage of the paranasal sinuses or the orbit.

Dx Test: CT scan
(which will also be needed to guide the surgery)
A 10 year old girl has epistaxis. Her mother says that she picks her nose all the time.
Dx?
Tx?
Dx: Bleeding from the Anterior part of the septum

Tx: Phenylephrine spray and local pressure
An 18 year old boy has epistaxis. The patient denies picking his nose. No source of anterior bleeding can be seen by physical examination.
Dx? (2 possible)
Dx:
1. Septal perforation from cocaine abuse
2. Posterior juvenile Nasopharyngeal Angiofibroma
A 72, hypertensive male, on aspirin for arthritis, has a copious nosebleed. His blood pressure is 220/105 when seen in the E.R. He says he began swallowing blood before it began to come out through the front of his nose.
Dx?
Management? (2)
Dx: Epistaxis secondary to hypertension

Management:
1. Lower BP with Medication
2. Involve ENT

(These are serious problems that can end up with death)
A 57 year old man seeks help for “dizziness”. On further questioning he explains that the room spins around him
Dx?
Management?
Dx: Vestibular Apparatus

Management: Symptomatic treatment (meclizine, phenergan, diazepam), or an ENT workup
During a school physical exam, a 12 year old girl is found to have a heart murmur. She is referred for further evaluation. An alert cardiology fellow recognized that she indeed has a pulmonary flow systolic murmur, but he also notices that she has a fixe
Dx: Atrial septal defect

Diagnostic test: Echocardiography

Tx: Surgical closure of the defect
A three month old boy is hospitalized for ‘failure to thrive”. He has a loud, pansystolic heart murmur best heard at the left sternal border. Chest X-Ray shows increased pulmonary vascular markings.
Dx?
Diagnostic test?
Tx?
Dx: Ventricular septal defect

Diagnostic test: Echocardiography

Tx: surgical correction
A three day old premature baby has trouble feeding and pulmonary congestion. Physical exam shows bounding peripheral pulses and a continuous, machinery-like heart murmur.
Dx?
Diagnostic test?
Tx? (2 possible)
Patent Ductus Arteriosus

Diagnostic test: Echocardiography

Tx:
1. Surgical closure
2. Indomethacin
A patient known to have a congenital heart defect requires extensive dental work.
Management?
Management: antibiotic prophylaxis for subacute bacterial endocarditis
A 6 year old boy is brought to the U.S. by his new adoptive parents, from an orphanage in Eastern Europe. The kid is small for his age, and has a bluish hue in the lips and tips of his fingers. He has clubbing and spells of cyanosis relieved with squatti
Dx: Tetralogy of Fallot

Diagnostic test: Echocardiogram
A 72 year old man has a history of angina and exertional syncopal episodes. He has a harsh midsystolic heart murmur best heard at the second intercostal space and along the left sternal border.
Dx?
Diagnostic test?
Definitive Tx?
When
Dx: Aortic Stenosis

Diagnostic test: Echocardiogram

Tx: Surgical Valvular replacement

Surgery indications:
1. gradient of more than 50 mm.Hg.
2. indication of CHF, angina or syncope
A 72 year old man has been known for years to have a wide pulse pressure and a blowing, high-pitched, diastolic heart murmur best heard at the second intercostal space and along the left lower sternal border with the patient in full expiration. He has ha
Dx: Chronic Aortic Insufficiency

Diagnostic test: Echocardiogram

Next step: Aortic valve replacement
A 26 year old drug-addicted man develops congestive heart failure over a short period of a few days. He has a loud, diastolic murmur at the right, second intercostal space. A physical exam done a few weeks ago, when he had attempted to enroll in a detoxi
Dx: Acute Aortic Insufficiency due to Endocarditis

Management:
1. Emergency valve replacement
2. Antibiotics for a long time
A 35 year old lady has dyspnea on exertion, orthopnea, paroxysmal nocturnal dyspnea, cough and hemoptysis. She has had these progressive symptoms for about 5 years. She looks thin and cachectic, has atrial fibrillation and a low-pitched, rumbling diastol
Dx: Mitral stenosis

Diagnostic test: Echocardiogram

Tx: Eventually surgical mitral valve repair
A 55 year old lady has been known for years to have mitral valve prolapse. She now has developed exertional dyspnea, orthopnea and atrial fibrillation. She has an apical, high pitched, holosystolic heart murmur that radiates to the axilla and back.
Dx: Mitral Regurgitation

Diagnostic test: Echocardiogram

Tx: eventually surgical repair of the valve (Annuloplasty) or possibly valve replacement
A 55 year old man has progressive, unstable, disabling angina that does not respond to medical management. His father and two older brothers died of heart attacks before the age of 50. The patient stopped smoking 20 years ago, but still has a sedentary l
It’s a heart attack waiting to happen...

Management: Cardiac Catheterization
(to see if he is a suitable candidate for coronary revascularization)
On a routine pre-employment physical examination, a chest X-Ray is done on a 45 year old chronic smoker. A “coin lesion” is found in the upper lobe of the right lung.
Dx?
Next step?
Dx: Cancer of the lung

Next step: Find and older chest X-Ray if one is available (from one or more years ago). If an older X-Ray has the same unchanged lesion, it is not likely cancer. No further work up is needed now, but the lesion should be followed with periodic X-Rays.
A 54 year old man with a 40 pack/year history of smoking gets a chest X-Ray because of persistent cough. A peripheral, 2cm “coin lesion” is found in the right lung. A chest X-Ray taken two years ago had been normal. CT scan shows no calcifications in
Dx: Cancer of the lung

Diagnostic test:
1. Start with Bronchoscopy and washings,
2. if unrewarding go to Percutaneous Needle Biopsy
3. if still unsuccessful go to Open Biopsy
(Thoracotomy and Wedge Resection)
A 72 year old chronic smoker with severe COPD is found to have a central, hilar mass on chest X-Ray. Bronchoscopy and biopsy establish a diagnosis of squamous cell carcinoma of the lung. His FEV1 is 1100, and a ventilation/perfusion scan shows that 60% o
Management: It takes an FEV1 of at least 800 to survive surgery and not be a pulmonary cripple afterwards. If this fellow got a pneumonectomy (which he would need for a central tumor) he would be left with an FEV1 of 440. No way... Don’t do any more tests. He is not a surgical candidate.

Tx: pursue Chemotherapy and Radiation
A 62 year old chronic smoker has an episode of hemoptysis. Chest X-ray shows a central hilar mass. Bronchoscopy and biopsy establish a diagnosis of squamous cell carcinoma of the lung. His FEV1 is 2200, and a ventilation/perfusion scan shows that 30% of
Diagnostic test: CT scan and Mediastinoscopy
(to ascertain if surgery has a decent chance to cure him)

Tx: Pneumonectomy
(can tolerate it due to high FEV1)
A 33 year old lady is undergoing a diagnostic work-up because she appears to have Cushing’s syndrome. Chest X-Ray shows a central, 3cm round mass on the right lung. Bronchoscopy and biopsy confirm a diagnosis of small cell carcinoma of the lung.
M
Management: Radiation and chemotherapy.

(Small cell lung cancer is not treated with surgery, and thus we have no need to determine FEV1 or nodal status)
A 54 year old right handed laborer notices coldness and tingling in his left hand as well as pain in the forearm when he does strenuous work. What really concerned him, though, is that in the last few episodes he also experienced transitory vertigo, blur
Dx: Subclavian Steal syndrome

(A combination of “claudication of the arm” with posterior brain neurological symptoms is classical for this)

Management: Angiographic study (If you had been given the vignette without it), then Vascular surgery
A 62 year old man is found on physical exam to have a 6cm pulsatile mass deep in the abdomen, between the xiphoid and the umbilicus
Dx?
Tx?
Dx: Abdominal Aortic Aneurysm

Tx: Elective surgical repair
A 62 year old man has vague, poorly described epigastric and upper back discomfort. He has been found on physical exam to have a 6cm pulsatile mass deep in the abdomen, between the xiphoid and the umbilicus. The mass is tender to palpation.
Dx?
Dx: Abdominal Aortic Aneurysm that is beginning to leak.

Management: Get a consultation with the vascular surgeons today
A 68 year old man is brought to the ER with excruciating back pain that began suddenly 45 minutes ago. He is diaphoretic and has a systolic blood pressure of 90. There is an 8cm pulsatile mass palpable deep in his abdomen, between the xiphoid and the umb
Dx: Abdominal Aortic Aneurysm, rupturing right now.

Tx: Emergency surgery
A retired businessman has claudication when walking more than 15 blocks.

Management?
Management: If he is smoking he should quit; otherwise he needs nothing

(Vascular surgery, or angioplasty and stenting are palliative procedures. They do not cure arteriosclerotic occlusive disease. Claudication has an unpredictable course, thus there is no advantage to an “early operation”)
A 56 year old postman describes severe pain in his right calf when he walks two or three blocks. The pain is relieved by resting 10 or 15 minutes, but recurs if he walks again the same distance. He can not do his job this way, and he does not qualify yet
Diagnostic test:
1. Start with Doppler studies
2. If he has significant gradient, Arteriogram comes next

Tx: Bypass surgery or stenting
A patient consults you because he “can not sleep”. On questioning it turns out that he has pain in the right calf, which keeps him from falling asleep. He relates that the pain goes away if he sits by the side of the bed and dangles the leg. His wife
Dx: Claudication

Dx test:
1. Start with Doppler studies
2. If he has significant gradient, Arteriogram comes next

Tx: Bypass surgery or stenting
A 45 year old man shows up in the ER with a pale, cold, pulseless, paresthetic, painful and paralytic lower extremity. The process began suddenly two hours ago. Physical exam shows no pulses anywhere in that lower extremity. Pulse at the wrist is 95 per
Dx: Embolization by the broken-off tail of a clot from the left atrium

Tx: Emergency surgery with use of Fogarty catheters to retrieve the clot
A 74 year old man has sudden onset of extremely severe, tearing chest pain that radiates to the back and migrates down shortly after it’s onset. His blood pressure is 220/100, he has unequal pulses in the upper extremities and he has a wide mediastinum
Dx: Dissecting aneurysm of the thoracic Aorta

Management:
1. if high BP, beta-blockers or IV nitrates to lower BP
(b/c forces that dissected the vessel plus the force of the dye injection could further shear the aorta)
2. Arteriogram
(first if BP is normal)

Tx:
Ascending Aorta = emergency surgery
Descending Aorta = intensive therapy (in the ICU) for the hypertension will be the preferable option.
A 62 year old right handed man has transient episodes of weakness in the right hand, blurred vision, and difficulty expressing himself. There is not associated headache, the episodes last about 5 or 10 minutes at the most, and they resolve spontaneously.
Dx: Transient Ischemic Attacks
in the territory of the left carotid artery
(probably an ulcerated plaque at the left carotid bifurcation)

Diagnostic test: Angiogram

Treatment: Carotid endarterectomy
A 61 year old man presents with a one year history of episodes of vertigo, diplopia, blurred vision, dysarthria and instability of gait. The episodes last several minutes, have no associated headache and leave no neurological sequela.
Dx?
Diagn
Dx: Transient Ischemic Attacks
(but now the vertebrals may be involved)

Diagnostic test: Arteriogram that examines all the arteries going to the brain (i.e. an aortic arch study)

Tx: Vascular surgery will follow
A 60 year old diabetic male presents with abrupt onset of right third nerve paralysis and contralateral hemiparesis. There was no associated headache. The patient is alert, but has the neurological deficits mentioned.
Dx?
Diagnostic test?
Dx: Stroke
(Neurological catastrophes that begin suddenly and have no associated headache are vascular occlusive)

Diagnostic test: CT scan

(Vascular surgery in the neck is designed to prevent strokes, not to treat them once they happen)

Deck Info

247

permalink